Last visit was: 26 Apr 2024, 17:55 It is currently 26 Apr 2024, 17:55

Close
GMAT Club Daily Prep
Thank you for using the timer - this advanced tool can estimate your performance and suggest more practice questions. We have subscribed you to Daily Prep Questions via email.

Customized
for You

we will pick new questions that match your level based on your Timer History

Track
Your Progress

every week, we’ll send you an estimated GMAT score based on your performance

Practice
Pays

we will pick new questions that match your level based on your Timer History
Not interested in getting valuable practice questions and articles delivered to your email? No problem, unsubscribe here.
Close
Request Expert Reply
Confirm Cancel
SORT BY:
Kudos
Tags:
Show Tags
Hide Tags
Intern
Intern
Joined: 08 Nov 2019
Posts: 7
Own Kudos [?]: 1 [0]
Given Kudos: 64
Send PM
Manager
Manager
Joined: 26 Apr 2016
Posts: 209
Own Kudos [?]: 48 [0]
Given Kudos: 6
GMAT 1: 640 Q44 V33
Send PM
Volunteer Expert
Joined: 16 May 2019
Posts: 3512
Own Kudos [?]: 6860 [0]
Given Kudos: 500
Manager
Manager
Joined: 14 Mar 2018
Posts: 82
Own Kudos [?]: 21 [0]
Given Kudos: 195
Location: India
Concentration: General Management, Finance
Schools: IIMA PGPX'22
GPA: 4
WE:Corporate Finance (Investment Banking)
Send PM
Re: For similar cars and drivers, automobile insurance for collision damag [#permalink]
HARRY113 wrote:
I go with A.
Since, The argument states that:
Insurance costs = G > F and # Collisions = F > G.
Therefore author might have assumed that Collision cost are less in G. Since, there is a possibility that, the collisions cost can be higher even if the # of collisions are less. So definitely the answer should be in that terms for the authors conclusion to be true.


what bearing does the information "insurance costs are higher in G than F" have on our analysis. Insurance costs are something that the car owners would have to born right? But when it comes to profit made by insurance companies in respective regions, then angle of cost would come into play right?

Because what confuses me is that what was the need at all but the author to provide us with the "For similar cars and drivers, automobile insurance for collision damage has always cost more
in Great port than in Fairmont."


Please, someone, respond. Proper explanations and analysis is appreciated.
Thanks,
MK
Senior Manager
Senior Manager
Joined: 09 Feb 2020
Posts: 385
Own Kudos [?]: 41 [0]
Given Kudos: 433
Location: India
Send PM
Re: For similar cars and drivers, automobile insurance for collision damag [#permalink]
nightblade354

If we changed option C to "Claim", G residents are more likely to claim than F residents. Then can we say this would be a correct answer?
Current Student
Joined: 31 Jul 2017
Status:He came. He saw. He conquered. -- Going to Business School -- Corruptus in Extremis
Posts: 1734
Own Kudos [?]: 5743 [0]
Given Kudos: 3054
Location: United States (MA)
Concentration: Finance, Economics
Send PM
Re: For similar cars and drivers, automobile insurance for collision damag [#permalink]
Expert Reply
krndatta wrote:
nightblade354

If we changed option C to "Claim", G residents are more likely to claim than F residents. Then can we say this would be a correct answer?


If this is the end result of the negation, then yes, per my initial explanation, that would be sufficient to ruin the argument.
Intern
Intern
Joined: 18 Sep 2020
Posts: 18
Own Kudos [?]: 14 [0]
Given Kudos: 126
GMAT Focus 1:
715 Q89 V86 DI81
Send PM
Re: For similar cars and drivers, automobile insurance for collision damag [#permalink]
I can deduce that option A is correct one only by looking at the conclusion. I want to know how the assumption is linking premise to the conclusion?
Director
Director
Joined: 17 Aug 2009
Posts: 627
Own Kudos [?]: 31 [0]
Given Kudos: 21
Send PM
For similar cars and drivers, automobile insurance for collision damag [#permalink]
How they make more profits. Profits = Revenue - cost. The cost of repair has to be less. That's what A says.
The option C is a strengthener (it makes us believe more in the conclusion). We are not looking here for a strengthener but for an assumption, a necessary condition.
B - the number of motorists doesn't matter out of scope.
D - Out of scope.
E - We don't care if they are already aware. Out of scope
GMAT Club Bot
For similar cars and drivers, automobile insurance for collision damag [#permalink]
   1   2 
Moderators:
GMAT Club Verbal Expert
6923 posts
GMAT Club Verbal Expert
238 posts
CR Forum Moderator
832 posts

Powered by phpBB © phpBB Group | Emoji artwork provided by EmojiOne